Brutal SC #35

This topic has expert replies
Master | Next Rank: 500 Posts
Posts: 175
Joined: Mon Jul 02, 2007 12:01 pm
Thanked: 15 times
Followed by:1 members

Brutal SC #35

by jangojess » Fri Sep 21, 2007 11:47 am
An economic recession can result from a lowering of employment rates triggered by a drop in investment, which causes people to cut consumer spending and starts a cycle of layoffs leading back to even lower employment rates.

A) a lowering of employment rates triggered by a drop in investment, which causes people to cut consumer spending and start a cycle of layoffs leading back to even lower employment rates.

B) a lowering of employment rates triggered by dropping investment, which causes people to cut consumer spending and starts a cycle of layoffs leading back to even lower employment rates.

C) falling employment rates triggered by a drop in investment, which cause cutbacks in consumer spending, starting a cycle of layoffs that lead to even lower employment rates.

D) falling employment rates that are triggered by a drop in investment, causing people to cut consumer spending and starting a cycle of layoffs that lead back to even lower employment rates.

E) falling employment rates that are triggered by a drop in investment, causing cutbacks in consumer spending and starting a cycle of layoffs leading to even lower employment rates.

OA : C
Trying hard!!!

Master | Next Rank: 500 Posts
Posts: 120
Joined: Sun Apr 13, 2014 3:55 am

by john83.amar » Sat Apr 19, 2014 7:53 am
C seems to be the best answer)

Junior | Next Rank: 30 Posts
Posts: 17
Joined: Fri Feb 14, 2014 10:05 am
Thanked: 2 times

by ajaysingh24 » Sun Apr 20, 2014 11:47 am
john83.amar wrote:C seems to be the best answer)
Why not d

GMAT/MBA Expert

User avatar
Elite Legendary Member
Posts: 10392
Joined: Sun Jun 23, 2013 6:38 pm
Location: Palo Alto, CA
Thanked: 2867 times
Followed by:511 members
GMAT Score:800

by [email protected] » Sun Apr 20, 2014 4:15 pm
Hi ajaysingh24,

Answer D includes some redundancies; by definition, a redundancy is unnecessary and is considered bad grammar.

Here are the redundant elements:

"falling underemployment rates THAT are triggered.....causing PEOPLE to cut consumer spending....layoffs that lead BACK to even lower...."

GMAT assassins aren't born, they're made,
Rich
Contact Rich at [email protected]
Image

Legendary Member
Posts: 774
Joined: Mon Jan 23, 2012 4:32 am
Thanked: 46 times
Followed by:14 members

by aditya8062 » Sun Apr 20, 2014 9:21 pm
hi rich
the question as it stands now can never have C as an answer
"which" in C has some serious problems
i feel that E is much better in given options

GMAT/MBA Expert

User avatar
GMAT Instructor
Posts: 2095
Joined: Tue Dec 04, 2012 3:22 pm
Thanked: 1443 times
Followed by:247 members

by ceilidh.erickson » Mon Apr 21, 2014 8:09 am
The differences between answer choices are subtle, and many come down to MEANING rather than grammar.

"a lowering of employment rates" implies that someone intentionally lowered them. This doesn't make sense - only "falling employment rates" makes sense. Eliminate A and B.

"causing people to cut consumer spending" also doesn't make sense. People can cut back on spending, but this phrasing makes it sound like an intentional choice to cut, rather than a result of some other cause. Eliminate D.

C versus E is a difficult split (and more hair-splitting than you're likely to see on the real GMAT).

aditya8062, be careful:
the question as it stands now can never have C as an answer
"which" in C has some serious problems
This is not strictly true. A clause beginning with "which" should modify the noun directly before it... with one exception. A "which" is allowed to hop over an essential modifier (such as a prepositional phrase or a past participle) to modify the noun that comes before that. For example:
Here is the basket of apples, which is heavier than I expected.

Here, the "which" is modifying "basket," not "apples."

In C, we're hopping over then entire phrase "triggered by..." to modify "rates." Stylistically this isn't great (the GMAT would rarely allow a "which" to hop quite so far), but technically it's not grammatically incorrect.

In E, the cause-effect structure is slightly reversed. The meaning of the sentence is that falling employment rates (which cause spending drops and layoffs) cause economic recessions.

In E, by using the adverbial modifiers "causing" and "starting," it implies that the "causing and starting" are the RESULTS of the clause "a recession can result from..." rather than the CAUSE. Adverbial modifiers often show cause/effect relationships:
I tripped, falling on my face.

So, there are several reasons (besides concision, which is rarely a deciding factor) that C is the more correct answer (though not necessarily a great one).
Ceilidh Erickson
EdM in Mind, Brain, and Education
Harvard Graduate School of Education

GMAT/MBA Expert

User avatar
GMAT Instructor
Posts: 2095
Joined: Tue Dec 04, 2012 3:22 pm
Thanked: 1443 times
Followed by:247 members

by ceilidh.erickson » Mon Apr 21, 2014 8:14 am
For a more GMAT-like use of the hopping-which rule, see:
https://www.beatthegmat.com/emily-dickin ... tml#563839
Ceilidh Erickson
EdM in Mind, Brain, and Education
Harvard Graduate School of Education

Legendary Member
Posts: 774
Joined: Mon Jan 23, 2012 4:32 am
Thanked: 46 times
Followed by:14 members

by aditya8062 » Mon Apr 21, 2014 8:46 am
ceilidh.erickson :In E, the cause-effect structure is slightly reversed. The meaning of the sentence is that falling employment rates (which cause spending drops and layoffs) cause economic recessions.

In E, by using the adverbial modifiers "causing" and "starting," it implies that the "causing and starting" are the RESULTS of the clause "a recession can result from..." rather than the CAUSE. Adverbial modifiers often show cause/effect relationships:
Ceilidh ,with all due respect to u ,i somehow feel that this interpretation is not right .i have read a post of Stacey koprince and RON in which they have specifically told that comma +verbing modifiers modify the previous clause (and not necessarily the previous main clause)

so in option E :An economic recession can result from falling employment rates that are triggered by a drop in investment, causing cutbacks in consumer spending and starting a cycle of layoffs leading to even lower employment rates.

the "causing cutbacks in consumer spending and starting a cycle of layoffs leading to even lower employment rates" will modify the subject "falling employment rates" plus the "that" clause and not "An economic recession can result"
kindly tell me if this new interpretation would make E a better answer
thanks and regards

Legendary Member
Posts: 774
Joined: Mon Jan 23, 2012 4:32 am
Thanked: 46 times
Followed by:14 members

by aditya8062 » Mon Apr 21, 2014 8:54 am
ceilidh.erickson :A "which" is allowed to hop over an essential modifier (such as a prepositional phrase or a past participle) to modify the noun that comes before that. For example:
thanks ceilidh .i agree that "which" modifier can hop through a prepositional phrase but can u please guide me to some official problem where "which" hops through "participle phrases"

thanks and regards